ChaseDream
搜索
返回列表 发新帖
查看: 1883|回复: 1
打印 上一主题 下一主题

SC問題急救!

[复制链接]
跳转到指定楼层
楼主
发表于 2011-8-6 06:08:34 | 只看该作者 回帖奖励 |倒序浏览 |阅读模式
1.An eyeglasses manufacturer tried to boost sales  for summer quarter by offering its distributors a special discount if their orders for that quarter exceeded those for last year's  summer quarter  by at least 20 percent. Many distributors qualified for this discount. Even with much merchandise discounted, sales increased enough to produce a healthy gain in net profits. The manufacturer plans to  repeat this success  by offering  the same sort of discount for the fall quarter.

Which of the following, if true, most clearly points to a flaw in the manufacturer's plan to repeat the successful performance of the summer quarter?
(a)The distributors most likely to qualify for the manufacturer's special discount are those whose orders were unusually low a year earlier.
(b)The distributors' ordering more goods in the summer quarter left them overstocked for the fall quarter.
正確答案是(b)  但是我選了(a) 敢問(a)到底哪裡出了錯呢?  如果證明了qualify last year 的manufacturer很少的話不就間接證明了net  profit 不是從他們身上來的嗎? 這樣不就point a flaw證明重複使用這策略無效?
收藏收藏 收藏收藏
沙发
发表于 2011-8-6 08:28:35 | 只看该作者
a说符合要求的distributors很多是去年没怎么订货的
那么活动的结果有两个
1. distributors还是不订货
2. 订货
但是从夏季来看,这个discount是有吸引力的,他们没有明显理由不订货。由于这些人去年没订货,那么这个计划肯定是成功的。

B给出了distributor明确不能订货的原因。
lz,这是个CR问题
您需要登录后才可以回帖 登录 | 立即注册

Mark一下! 看一下! 顶楼主! 感谢分享! 快速回复:

IESE MBA
近期活动

正在浏览此版块的会员 ()

手机版|ChaseDream|GMT+8, 2024-10-6 12:25
京公网安备11010202008513号 京ICP证101109号 京ICP备12012021号

ChaseDream 论坛

© 2003-2023 ChaseDream.com. All Rights Reserved.

返回顶部